help w−3/4=8 a) 5 b) 35 c) 29 d) -1

Answers

Answer 1

Answer:

w = 35/4

im not exactly sure why the options are whole numbers.

Step-by-step explanation:

8 = w-3/4

-w + 8 = -3/4

-w = -3/4 - 8

-w = -3/4 - 32/4

-w = -35/4

w = 35/4


Related Questions

the slope of a line parallel to the given line 8x-2y=5

Answers

Answer:

4x

Step-by-step explanation:

8x-2y=5

8x=2y+5

8x-5=2y

4x-5/2=y

The slope of the parallel line would be 4x because the slope doesn't change. Hope this helps.

Solve for x in the equation x squared + 11 x + StartFraction 121 Over 4 EndFraction = StartFraction 125 Over 4 EndFraction.

Answers

Answer:

Below

Step-by-step explanation:

● x^2 + 11x + 121/4 = 125/4

Substract 125/4 from both sides:

● x^2 + 11x + 121/4-125/4= 125/4 -125/4

● x^2 + 11x - (-4/4) = 0

● x^2 +11x -(-1) = 0

● x^2 + 11 x + 1 = 0

This is a quadratic equation so we will use the determinanant (b^2-4ac)

● a = 1

● b = 11

● c = 1

● b^2-4ac = 11^2-4*1*1 = 117

So this equation has two solutions:

● x = (-b -/+ √(b^2-4ac) ) / 2a

● x = (-11 -/+ √(117) ) / 2

● x = (-11 -/+ 3√(13))/ 2

● x = -0.91 or x = -10.9

Round to the nearest unit

● x = -1 or x = -11

The solutions are { -1,-11}

The solution of the equation x² + 11x + (121/4) = 125/4 will be 0.09 and negative 11.09.

What is the solution to the equation?

The distribution of weights to the variables involved that establishes the equilibrium in the calculation is referred to as a result.

The equation is given below.

x² + 11x + (121/4) = 125/4

Simplify the equation, then the equation will be

4x² + 44x + 121 = 125

4x² + 44x + 121 - 125 = 0

4x² + 44x - 4 = 0

x² + 11x - 1 = 0

We know that the formula, then we have

[tex]\rm x = \dfrac{-b \pm \sqrt {b^2 - 4ac}}{2a}[/tex]

The value of a = 1, b = 11, and c = -1. Then we have

[tex]\rm x = \dfrac{-11 \pm \sqrt {11^2 - 4 \times 1 \times (-1)}}{2 \times 1}\\\rm x = \dfrac{-11 \pm \sqrt {121 +4}}{2 }\\x = \dfrac{-11 \pm \sqrt {125}}{2 }[/tex]

Simplify the equation, then we have

x = (- 11 ± 11.18) / 2

x = (-11 - 11.18) / 2, (-11 + 11.18) / 2

x = -11.09, 0.09

The solution of the equation will be 0.09 and negative 11.09.

More about the solution of the equation link is given below.

https://brainly.com/question/545403

#SPJ6

PLEASE HELP!!!!!! URGENT!!!!!!!!! ALGEBRA! Hannah, Yu, and Becky went to buy groceries together. All three of them bought pasta, noodles, and split peas. Hannah bought 3 packs of pasta, 10 packs of noodles, and 2 packs of split peas. Yu bought 5 packs of pasta, 12 packs of noodles, and 1 pack of split peas. Becky bought 3 packs of pasta, 15 packs of noodles, and 2 packs of split peas. Hannah, Yu, and Becky spent $8.41, $9.42, and $9.66 respectively on these items. What are the costs of a pack of pasta, a pack of noodles, and a pack of split peas? A) Pasta: $0.69 per pack, Noodles: $0.25 per pack, Split peas: $0.99 per pack B) Pasta: $1.09 per pack, Noodles: $0.20 per pack, Split peas: $1.97 per pack C) Pasta: $0.99 per pack, Noodles: $0.25 per pack, Split peas: $1.47 per pack D) Pasta: $0.69 per pack, Noodles: $0.20 per pack, Split peas: $1.97 per pack

Answers

Answer: C

Step-by-step explanation:

The costs of a pack of pasta, a pack of noodles, and a pack of split peas are Pasta: $0.99 per pack, Noodles: $0.25 per pack, Split peas: $1.47 per pack

PLEASE ANSWER QUICKLY ASAP
ANSWER QUESTION A​

Answers

Answer:

2934.46692

Step-by-step explanation:

if you need to round it,

hundrenths: 2,934.47

tenths: 2,934.5

Thousandths: 2,934.467

Hope this helps!

simpily 2^3×3^2=6^5​

Answers

Answer:

2^3×3^2=6^5​  equation is wrong because

2×2×2×3×3=72

6^5=6×6×6×6×6=36×36×6=7776

the two numbers are not equal

Mate, I think your question is wrong ! ;(

[tex]Corrected \\ Question...\\[/tex] (2^3)^2*(3^2)^3=6^5

How can you change a rational number to a decimal? Can you give an exsample?

Answers

Answer:

1/2=0.5

Step-by-step explanation:

¼=0.25

¾=0.75

what is the answer to 1/8=s-3/4

Answers

Answer:

7/8 =s

Step-by-step explanation:

1/8=s-3/4

Add 3/4

1/8 + 3/4 = s -3/4 +3/4

1/8 + 3/4 = s

Get a common denominator

1/8 + 3/4 *2/2 = s

1/8 + 6/8 =s

7/8 =s

1/8 = s - 3/4

1/8 = s -6/8 ( * 2/2)

7/8 = s

s = 7/8

Joey intends to roll a six-sided number cube 100 times. What probability model can he use to predict whether or not each roll will give a result that is divisible by 3?

Answers

Options :

A. Each roll has a 0.117 probability of being divisible by 3.

B. Each roll has a 0.333 probability of being divisible by 3.

C. Each roll has a 0.5 probability of being divisible by 3. D. Each roll has a 0.667 probability of being divisible by 3.

Answer: B. Each roll has a 0.333 probability of being divisible by 3.

Step-by-step explanation:

Sample space for a six-sided number cube :

1, 2, 3, 4, 5, 6

Number of outcomes divisible by 3:

(3, 6) = 2

Probability of an event = Number of required outcomes / total number of possible items

Probability (getting a number divisible by 3):

(Number of outcomes divisible by 3 / total outcomes in sample space)

Probability (getting a number divisible by 3):

2 / 6 = 1/3

= 0.333

If sin Θ = 5 over 6, what are the values of cos Θ and tan Θ?

Answers

Answer:

Check explanation

Step-by-step explanation:

Sin∅=5/6

Opp=5. Hyp=6

Adj= (√6²+5²)

= √11

Cos∅=(√11)/6

Tan∅=5/(√11)

Answer: cos(theta) = sqrt(11)/6 ; tan(theta)=5/sqrt(11).

Explanation:
sin inverse is restricted to the first and fourth quadrant, so the other side of the triangle is positive sqrt(11) by the Pythagorean theorem.

Find the probability of drawing 3 Aces at random from a deck of 52 ordinary playing cards if the cards are:_________
A) Replaced
B) Not Replaced

Answers

Answer:

a. With replacement

1/2197

b. Without replacement

1/5,525

Step-by-step explanation:

Okay, here is a probability question.

The key to answering this question is by knowing the number of aces in a deck of cards.

There is 1 ace per suit, so there is a total of 4 aces per deck of cards.

So, mathematically the probability of picking an ace would be;

number of aces/ total number of cards = 4/52 = 1/13

a. Now since the action is with replacement; that means that at any point in time, the total number of cards would always remain 52 even after making our picks.

So the probability of picking three aces with replacement would be;

1/13 * 1/13 * 1/13 = 1/2197

b. Without replacement

what this action means is that after picking a particular card, we do not return the picked card to the deck of cards.

For the first card picked, we will be having a total of 4 aces and 52 total cards.

So the probability of picking an ace would be 4/52 = 1/13

For the second card picked, we shall be left with selecting an ace out of the remaining 3 aces and the total remaining 51 cards

So the probability will be 3/51 = 1/17

For the third and last card to be picked, we shall be left with picking 1 out of the remaining 2 aces cards and out of the 50 cards left in the deck.

So the probability now becomes 2/50 = 1/25

Thus, the combined probability of picking 3 aces cards without replacement from a deck of cards will be;

1/13 * 1/17 * 1/25 = 1/5,525

Using the binomial and the hypergeometric distribution, it is found that the probabilities are:

a) 0.0005 = 0.05%.

b) 0.0002 = 0.02%.

Item a:

With replacement, hence the trials are independent, and the binomial distribution is used.

Binomial probability distribution

[tex]P(X = x) = C_{n,x}.p^{x}.(1-p)^{n-x}[/tex]

[tex]C_{n,x} = \frac{n!}{x!(n-x)!}[/tex]

The parameters are:

x is the number of successes. n is the number of trials. p is the probability of a success on a single trial.

For this problem:

In a deck, there are 52 cards, of which 4 are Aces, hence [tex]p = \frac{4}{52} = 0.0769[/tex]3 cards are drawn, hence [tex]n = 3[/tex].

The probability is P(X = 3), then:

[tex]P(X = x) = C_{n,x}.p^{x}.(1-p)^{n-x}[/tex]

[tex]P(X = 3) = C_{3,3}.(0.0769)^{3}.(0.9231)^{0} = 0.0005[/tex]

0.0005 = 0.05% probability.

Item b:

Without replacement, hence the trials are not independent and the hypergeometric distribution is used.

Hypergeometric distribution:

[tex]P(X = x) = h(x,N,n,k) = \frac{C_{k,x}C_{N-k,n-x}}{C_{N,n}}[/tex]

[tex]C_{n,x} = \frac{n!}{x!(n-x)!}[/tex]

The parameters are:

x is the number of successes. N is the size of the population. n is the size of the sample. k is the total number of desired outcomes.

In this problem:

Deck of 52 cards, hence [tex]N = 52[/tex].4 of the cards are Aces, hence [tex]k = 4[/tex].3 cards are drawn, hence [tex]n = 3[/tex].

The probability is also P(X = 3), hence:

[tex]P(X = x) = h(x,N,n,k) = \frac{C_{k,x}C_{N-k,n-x}}{C_{N,n}}[/tex]

[tex]P(X = 3) = h(3,52,3,4) = \frac{C_{4,3}C_{48,0}}{C_{52,3}} = 0.0002[/tex]

0.0002 = 0.02% probability.

To learn more about the binomial and the hypergeometric distribution, you can take a look at https://brainly.com/question/25783392

4x=24 solve equation

Answers

Answer:

x=6

Step-by-step explanation:

Rearrange:

Rearrange the equation by subtracting what is to the right of the equal sign from both sides of the equation :

                    4*x-(24)=0

Step by step solution :

STEP

1

:

Pulling out like terms

1.1     Pull out like factors :

  4x - 24  =   4 • (x - 6)

Equation at the end of step

1

:

STEP

2

:

Equations which are never true

2.1      Solve :    4   =  0

This equation has no solution.

A a non-zero constant never equals zero.

Solving a Single Variable Equation:

2.2      Solve  :    x-6 = 0

Add  6  to both sides of the equation :

                     x = 6

One solution was found :

x = 6

Answer:

x= 24/ 4

Step-by-step explanation:

You can simplify it

x= 6/1 which is x= 6

PLEASE HELP, WILL GIVE BRAINLIEST IF CORRECT!!!! (08.06 MC) Mike and his friends bought cheese wafers for $2 per packet and chocolate wafers for $1 per packet at a carnival. They spent a total of $25 to buy a total of 20 packets of wafers of the two varieties. Part A: Write a system of equations that can be solved to find the number of packets of cheese wafers and the number of packets of chocolate wafers that Mike and his friends bought at the carnival. Define the variables used in the equations. (5 points) Part B: How many packets of chocolate wafers and cheese wafers did they buy? Explain how you got the answer and why you selected a particular method to get the answer. (5 points)

Answers

Answer:

x = 5 , y = 15

Step-by-step explanation:

You can solve this using substitution.

Let the quantity of cheese wafers be denoted by x and the quantity of chocolate wafers denoted by y

2x + 1y = 25

x + y = 20

These two equations are the answer to part A, (remember to include the above prompt which says what x and y denote).

For part B I used substitution because it was more applicable to the question then addition or elimination.

ACTUAL WORK

Set 2x + 1y = 25 equal to x

x = 25 - y / 2

Replace x with y in the second equation

(25 - y / 2) + y = 20

And solve for y

y = 15

Since we know what y is we can replace y in the second equation and find what x is

x + 15 = 20

Solve for x

x = 5

Answer:

5 Cheese Wafers and 15 Chocolate Wafers

Step-by-step explanation:

Tim has an after-school delivery service that he

provides for several small retailers in town. He

uses his bicycle and charges $1.25 for a delivery

made within 1 mi, $1.70 for a delivery of at

least 1 mi but less than 1 mi, $2.15 for a

delivery of at least 1. mi but less than 2 miles,

and so on. If Tim raised his rates by 10%, what

would he be paid to deliver a package 35

miles?

Answers

Answer:

Step-by-step explanation:

tim has an after school delivery service that he provides for several small retailers in town. he uses his bicycle and charges $1.25 for a delivery made within 1 1/2 miles, $1.70 for a delivery of at least 1 1/2 miles but less than 1 3/4 miles. $2.15 for a delivery of at least 1 3/4 miles but less than 2 miles, and so on. if tim raised his rates by 10%, what would he be paid to deliver a package 3 1/8 miles.

Answer:

From the question asked the cost of additional 1/4 mile (1 3/4 - 1 1/2) is $0.45 ($1.7 - $1.25). If the rate is increased by 10% (0.1), the new price for an additional 1/4 mile would be 1.1 (1 + 0.1) × 0.45 = $0.495.

Tim new charge rate are as follows:

$1.25 for a delivery made within 1 1/2 miles

$1.745 for a delivery of at least 1 1/2 miles but less than 1 3/4

$2.24 or a delivery of at least 1 3/4 miles but less than 2

$2.735 or a delivery of at least 2 miles but less than 2 1/4

$3.23 or a delivery of at least 2 1/4 miles but less than 2 1/2

$3.725 or a delivery of at least 2 1/2 miles but less than 2 3/4

$4.22 or a delivery of at least 2 3/4 miles but less than 3

$4.715 or a delivery of at least 3 miles but less than 3 1/4

Since 3 1/8 is within 3 miles and 3 1/4 miles, Tim would charge $4.715 to deliver a package 3 1/8 miles.

 

need help will give 5 stars.

Answers

Answer:

t=0.64

Step-by-step explanation:

h = -16t^2 +4t +4

We want h =0 since it is hitting the ground

0 = -16t^2 +4t +4

Using the quadratic formula

a = -16  b = 4  c=4

-b ± sqrt( b^2 -4ac)

----------------------------

         2a

-4 ± sqrt( 4^2 -4(-16)4)

----------------------------

         2(-16)

-4 ± sqrt( 16+ 256)

----------------------------

         -32

-4 ± sqrt( 272)

----------------------------

         -32

-4 ± sqrt( 16*17)

----------------------------

         -32

-4 ± sqrt( 16) sqrt(17)

----------------------------

         -32

-4 ± 4 sqrt(17)

----------------------------

         -32

Divide by -4

1 ±  sqrt(17)

----------------------------

         8

To the nearest hundredth

t=-0.39

t=0.64

Since time cannot be negative

t=0.64

Answer:

0.64  

Step-by-step explanation:

0 = -16t^2 + 4t + 4

-4(4t^2 - t -1) = 0

t = [-(-1) +/- sqrt (1 - 4*4*-1)] / 8)

t = 0.64, -0.39

answer is 0.64

Victor fue al mercado para comprar manzanas, naranjas y platanos; las naranjas costaron el doble de lo 1ue pago por las manzanas y los platanos costaron 8 pesos menos que pas manzanas, en total gasto 100 pesos. Determina el precio de las manzanas, naranjas y platanos

Answers

Answer:

El precio de las manzanas = 27 pesos

El precio de las naranjas = 54 pesos

El precio de las bananas = 19 pesos

Step-by-step explanation:

Los parámetros dados son;

El monto total gastado = 100 pesos

Sea el precio de las naranjas = x

Sea el precio de las manzanas = y

Sea el precio de los plátanos = z

La cantidad pagada por las naranjas = 2 · y = x

La cantidad pagada por los plátanos = y - 8 = z

Por lo tanto, tenemos;

La cantidad total gastada = La cantidad pagada por las naranjas + La cantidad pagada por las bananas + La cantidad pagada por las manzanas

∴ El monto total gastado = 100 pesos = 2 · y + y - 8 + y

100 = 4 · años - 8

4 · y = 100 + 8 = 108

y = 108/4 = 27

y = 27

De

z = y - 8 tenemos;

z = 27 - 8 = 19

De 2 · y = x, tenemos;

2 × 27 = x

x = 54

Por lo tanto;

El precio de las naranjas = 54 pesos

El precio de las manzanas = 27 pesos

El precio de los plátanos = 19 pesos.

find six rational numbers between 3 and 4​

Answers

13/4, 10/3, 7/2, 11/3, 15/4, 35/9

Answer:

3.1

3.2

3.3

3.4

3.5

3.6

Hope this answer correct :)

Evaluate the following expression. −8 × (−10) −7× 1/−1

Answers

Answer:

87

Step-by-step explanation:

[tex]-8\left(-10\right)-7 \times \frac{1}{-1}=87\\\\\mathrm{Apply\:rule}\:-\left(-a\right)=a\\\\=8\times \:10-7\times \frac{1}{-1}\\\\8\times \:10=80\\\\7\times \frac{1}{-1}=-7\\\\=80-\left(-7\right)\\\\\mathrm{Apply\:rule}\:-\left(-a\right)=a\\\\=80+7\\\\=87[/tex]

acute angle between the hours hand and the minute hand at 1pm​

Answers

Answer: 30 degrees

Step-by-step explanation:

1 hour = 60 min = 360 degree

1 min = 360/60 degree

1 min = 6 degree

and the gap of hour hand and minute hand at 1pm, is of 5 min

therefore acute angle formed is 5 X 6 = 30 degrees.

:-)

Answer:

30 degrees

Step-by-step explanation:

1 hour = 60 min = 360 degree

1 min = 360/60 degree

1 min = 6 degree

and the gap of hour hand and minute hand at 1pm, is of 5 min

therefore acute angle formed is 5 X 6 = 30 degrees.

A bag contains twelve marbles, which includes seven red marbles and five blue marbles. Roja reaches into the bag and pulls out four marbles. a) How many different sets of four marbles can be pulled from this bag? b) How many of these sets contain two red marbles and two blue marbles? c) How many of these sets contain all red marbles? d) How many of these sets contain all red marbles or all blue marbles?

Answers

Answer:

a) 495

b) 210

c) 35

d) 40

Step-by-step explanation:

Given a total of 12 marbles.

n = 12

Number of red marbles = 7

Number of blue marbles = 5

a) Number of different sets of 4 marbles that can be made from this bag ?

This is a simple combination problem.

where n = 12 and r = 4.

So, answer will be:

[tex]_{12}C_4[/tex]

Formula:

[tex]_{n}C_r = \dfrac{n!}{(n-r)!r!}[/tex]

[tex]_{12}C_4 = \dfrac{12!}{(8)!4!} = \dfrac{12\times 11\times 10\times 9}{4 \times 3\times 2} =\bold{495}[/tex]

b) Two red and two blue marbles:

The answer will be:

[tex]_{7}C_2 \times _{5}C_2 = \dfrac{7\times 6}{2} \times \dfrac{5\times 4}{2} =\bold{210}[/tex]

c) all red marbles.(4 chosen out of 7 red and 0 chosen out of 5 blue marbles)

[tex]_{7}C_4 \times _{5}C_0 = \dfrac{7\times 6\times 5\times 4}{4\times 3\times 2} =\bold{35}[/tex]

d) all red or all blue.(all red marbles plus all blue marbles)

All red marbles:

[tex]_{7}C_4 \times _{5}C_0 = \dfrac{7\times 6\times 5\times 4}{4\times 3\times 2} \times 1=\bold{35}[/tex]

All blue marbles:

[tex]_{7}C_0 \times _{5}C_4 = 1 \times \dfrac{5\times 4\times 3\times 2}{4\times 3\times 2} =\bold{5}[/tex]

So, answer is 40.

Can someone please help!! ****This problem is multiple choice!

Answers

Answer:

I believe it's A) 302.94

Step-by-step explanation:

There are 360 degrees in a circle. So I enetered 360/112 in a calculator and got 3.214.

Then I multiplied 3.214 by 30pi and got 302.939 AKA 302.94. I did this because I looked at it like a ratio problem but idk if thats correct.

This answer should be letter 302.94 A.

Evaluate the function below at x=5. Then, enter your solution. f(x)=3(2)^x

Answers

Answer:

Solution: f(5) = 96

Step-by-step explanation:

f(5) = 3(2)^5

f(5) = 3 (2 × 2 × 2 × 2 × 2)

f(5) = 3 (32)

f(5) = 96

4 The surface area of a cube with side s is A = 682
Use the formula to find the surface area of a cube with s=4.

Answers

Answer:

96

Step-by-step explanation:

SA = 6*s^2

Since s = 4, we need to square it.

4^2 (4 Squared) = 16

Now we need to multiply 16 by 6 since there are 6 sides on a square.

16*6 = 96

So our Answer is 96.

--Variables

SA= Surface Area

s = Side Length or 4

a blue die and a green die are rolled. find the probability that the blue and green are both less than 6​

Answers

Answer

5/6 maybe

Step-by-step explanation:

Evaluate f(x) = 2|x – 5| for f(–5) and f(0).

Question 20 options:

f(–5) = –20, f(0) = –2

f(–5) = 20, f(0) = 10

f(–5) = 10, f(0) = 0

f(–5) = 12, f(0) = 5

Answers

Answer:

[tex]\Large \boxed{\mathrm{f(-5) = 20, \ f(0) = 10}}[/tex]

Step-by-step explanation:

The function is given:

f(x) = 2|x - 5|

Solve for f(-5).

x = -5

f(-5) = 2|-5 - 5|

f(-5) = 2|-10|

f(-5) = 2(10) = 20

Solve for f(0).

x = 0

f(-5) = 2|0 - 5|

f(-5) = 2|-5|

f(-5) = 2(5) = 10

Need help please asap this is not asap but please still give an answer im stuck

Answers

Answer:

135 cubes

Step-by-step explanation:

First, find the volume of the box with the equation V = Bh, where B is the area of the base and h is the height.

V = (2.25)(0.75)(1.25)

V = 2.109375

Next, find the volume of one cube with the side length 1/4 with V = Bh:

V = (0.25)(0.25)(0.25)

V = 0.015625

Then, divide the volume of the box by the volume of one cube:

2.109375 / 0.015625

= 135

A rectangular prism has a volume of 864 cubic units. How many cubic unit will fill the volume of the solid if they were packed without any gaps or overlaps

Answers

Answer: 864.

Step-by-step explanation:

The volume of a rectangular prism has a volume equal to:

V = W*L*H

W = width

L = length

H = height

We know that the volume is equal to 864 cubic units.

This means that if we want to fill the prism such that there is no gap or overlap, we should use exactly 864 unit cubes.

Nancy needs at least 1000 gigabytes of storage to take pictures and videos on her upcoming vacation. She
checks and finds that she has 105 GB available on her phone. She plans on buying additional memory
cards to get the rest of the storage she needs.
The cheapest memory cards she can find each hold 256 GB and cost $10. She wants spend as little money
as possible and still get the storage she needs.
Let C represent the number of memory cards that Nancy buys.

Answers

Answer:

C = 4 memory cards.

Step-by-step explanation:

256 × 4 = 1024

1024 + 105 = 1129 GB

She needs 4 memory cards.

Nancy needs to buy 4 memory cards.

Given that Nancy needs at least 1000 gigabytes of storage to take pictures and videos on her upcoming vacation, and she checks and finds that she has 105 GB available on her phone, and she plans on buying additional memory cards to get the rest of the storage she needs, and the cheapest memory cards she can find each hold 256 GB and cost $ 10, and she wants to spend as little money as possible and still get the storage she needs, to determine how many memory cards to buy, the following calculation must be performed:

(1000 - 105) / 256 = C 895/256 = C 3.49 = C

So if Nancy buys 3 cards she will still be short on gigabytes. Therefore, she must buy 4 memory cards.

Learn more in https://brainly.com/question/9154717

please can someone help me solve this.. please help!!​

Answers

Step-by-step explanation:

Hello,

Firstly just look to triangle BDE,

Here, you will find that,

140° = y+80° {the exterior and opposite interior angle of a triangle is equal}.

or, y= 140°-80° {shifting 80° to another side and subtracting it.}

Therefore, the value of y is 60°.

now, let's simply work with line EB or EG. we get;

angle GEF + y=180° { being a linear pair}.

or, angle GEF + 60°= 180°

or, angle GEF = 180°-60°

Therefore, the value of angle GEF = 120°.

now, looking in triangle EFG, we get;

angle GEF + 35°+x= 180° { the sum of interior angle of a triangle is 180°}.

or, 120°+35°+ x= 180°

or, x= 180°- 155°

Therefore, the value of x is 25°.

now, lastly finding the value of "z"

We find that x= z {being vertical opposite angle}

or, z =25°

Therefore, the value of z is 25°.

So, the values are,

x=25°

y=60°

and z= 25°

Hope it helps...

Some time ago , Keith's height and his nephew's height were at a ratio of 15:7. Then, Keiths height increased by 16% and his nephew,s height doubled. Keith is now 34 cm taller than his nephew, what is their total current height

Answers

Answer:

The answer is below

Step-by-step explanation:

The ratio of Keith's height and his nephew's height is 15:7. Let keith height be x cm and his nephews height be y cm.

[tex]\frac{x}{y}=\frac{15}{7} \\x=\frac{15}{7}y[/tex]

Keiths height increased by 16% , therefore Keith new height is (100% + 16%) × x = 1.16x

The nephew height is doubled, therefore his new height is 2y.

Given that Keith is now 34 cm taller than his nephew

1.16x = 2y + 34

but x = (15/7)y

[tex]1.16(\frac{15}{7} )y=2y+34\\\\\frac{87}{35} y=2y+34\\\\\frac{87}{35} y-2y=34\\\\\frac{17}{35}y=34\\ \\y=\frac{34*35}{17}\\ \\y=70\ cm[/tex]

The nephews new height = 2y = 2(70) = 140 cm

Keith new height = 2y + 34 = 140 + 34 = 174 cm

Their total current height = 140 cm + 174 cm = 314 cm

Which operation involving complex numbers requires the use of a conjugate to be carried​ out?

Answers

Answer:

The correct answer will be "Division".

Step-by-step explanation:

The procedure represents numerous values requiring something like a conjugate to have been done becomes division, since the denominator conjugate multiplies the numeric values including its quotient to represent the quotient of several complex numbers throughout the standard language.It is indeed a method used to separate a set of items across equal proportions.
Other Questions
Determine the area of the shape above. The formula for the area of a polygon is: Area = 1/2 (a n s) * On a map, two locations are 0.75 centimeter apart. Their actual distance is 15 kilometers apart. What scale could beshown on the map? Select three options. If i have 1 meerkat and you have 2, how many meerkats in total? A ladder leans against a vertical at angle of 60 to the wall of the foot of the ladder is 5m away from the wall calculate the length of the ladder According to the Navigation Rules, a risk of imminent collision exists in which situation? Boats are operating in open waters less than 40 feet from each other A vessel is exiting an area of commercial vessels A vessel is operating in a marine sanctuary The course of an approaching vessel does not change An engineer wishes to determine the width of a particular electronic component. If she knows that the standard deviation is 3.6 mm, how many of these components should she consider to be 90% sure of knowing the mean will be within 0.1 0.1 mm? Which temperature during the week is the closest to 0? Explain how you decided this using absolute value.Which temperature during the week is farthest from 0? Explain how you decided this using absolute value.Which is the coldest temperature? Explain how you determined this using the number line.Which is the warmest temperature? Explain how you determined this using the number line.Part 2: Weather ReportUse the information from the analysis and develop a weather report. You can develop a flyer or make a presentation, video or audio recording of yourself, or script. Use your creativity! Make sure to include the information from all six questions. Why did the Greeks resist the Persians how long did Neanderthal live All of the following actions by a custodian in an account opened under the Uniform Gifts to Minors Act are permitted except:_______. A. donating funds to the account to make additional investments B. withdrawing funds from the account for the custodian's use C. managing the investments in the account with the objective of generating enough income for college tuition D. selling securities in the account to generate proceeds for other investments vitamin c is obtained_____ citrus fruitsA. fromB. ofC. inD. by 85 POINTS! PLEASE HELP! Explain how to write an equation parallel to the equation y = 2x + 3 and the new line also includes the ordered pair (1,-2). Compute using long division: 1,23468 As part of your research, you examine mRNA expression for the parents, their two unaffected children, and their affected child with a Northern blot using a full-length cDNA probe.Your results, shown here, are as follows:Both parents have low, but detectable, mRNA expression.The unaffected children have either normal or low mRNA expression.The affected child has no detectable mRNA expression. What possible molecular defect could explain these results?Select every legitimate possibility from the choices below.a. The recessive allele could be a premature stop codon that prevents the translation of a full-length protein.b. The recessive allele could be a mutation in the promoter region that prevents RNA polymerase II from binding.c. The recessive allele could be a mutation in the transcribed region that makes the mRNA unstable and causes it to degrade.d. The recessive allele could be a mutation in regulatory DNA that prevents chromatin from being remodeled from a closed to an open state.e. The recessive allele could be a mutation in a splice donor or acceptor sequence that prevents proper mRNA splicing, leaving an intron in the spliced transcript. Find the value of x. A. 45 B. 30 C. 6 D. 15 You have an option on an ARM that has a two-year adjustment interval, 4% margin, 2% periodic rate cap and a 6% lifetime cap. The current initial rate is 5.35%. You can also get a 30-year, fixed-rate mortgage for 6.65%. You plan on staying in this home for at least 10 years. What would be your best choice? Why did many oppose the national bank?The bank would not pay for the national debt.The bank could not provide mortgages.A national bank was not mentioned in the Constitution. Type the correct answer in each box. If necessary, use / for the fraction bar.Complete the statements about series A and B.Series A: 10 + 4 +8/5 + 16/25 + 13/125 + ...Series B: 1/5 + 3/5 + 9/5 + 27/5 + 81/5 +...Series (a or b) has an r value of (blank) where 0 Rima eat guava explain how the teeth tongue and sliva will help to break it down Helpppppppppppppppp plzzz